Download as pdf or txt
Download as pdf or txt
You are on page 1of 20

1. A client as an obsessive-compulsive disorder b. 0.

5 tablet
manifested by the compulsion of handwashing. The c. 4 tablets
nurse knows that which of the following best d. 1 tablet
describes the client’s need for the repetitive acts of
handwashing? 9. A client is admitted with suspected pneumonia. The
a. Handwashing represents an attempt to chest xray reveals right middle and lower lunch
manipulate the environment to make it consolidation. During auscultation of the middle and
more comfortable. lower lobes, which finding related to the pulmonary
b. Handwashing externalizes the anxiety from system would the nurse anticipate?
a source within the bidy to an acceptable a. Inspiratory and expiratory wheezing
substitute outside the body. b. Decreased breath sounds
c. Handwashing assists the client to avoid c. Tympanic hyperresonance
undesirable thoughts and maintain some d. Bronchovesicular sounds
control over guilt and anxiety.
d. Handwashing helps to maintain the client in 10. Prior to electroconvulsive therapy (ECT) treatment,
an active state to resist the effects of the patient receives an injection of a medication
depression. that reduces secretions and protects against vagal
bradycardia. Which medication will you administer?
2. Following the vaginal delivery of an 11-pound baby, a. Fluoxetine (Prozac)
the nurse encourages the mother to breastfeed her b. Diphenhydramine (Bendaryl)
newborn. What is the primary purpose of this action? c. Atropine
a. To initiate the secretion of colostrum d. Epinephrine (Adrenalin)
b. To prevent neonatal hyperglycemia
c. To facilitate maternal-newborn interaction. 11. Patient with Alzheimer’s wandering in the hallway,
d. To stimulate the uterus to contract. which of the following should the nurse do?
a. Place in soft restraints
3. Client is admitted for a series of tests to verify the b. Place in a restraint chair in the nurse
diagnosis of Cushing’s syndrome. Which of the station
following assessment findings, if observed by the c. Place chemical restraints
nurse, would support this diagnosis? d. Ask relative to stay with the patient
a. Buffalo hump, hyperglycemia, and
hypernatremia 12. Mrs. Robinson is a 38 year old woman being treated
b. Nervousness, tachycardia, and intolerance on an outpatient basis for depression. Three months
to heat ago, her husband revealed that he was having an
c. Lethargy, weight gain and intolerance to affair with her best friend and planned to file for
cold divorce. Three weeks ago, Mrs. Robinson’s 14 year
d. Irritability, moon face and dry skin old son (her only child) committed suicide on an
inpatient psychiatric mental health unit.in today’s
4. The nurse is caring for a child with acute renal therapy session, Mrs. Robinson reveals to her nurse
failure who is in the intensive care unit. Which therapist that she is seriously contemplating suicide
assessment finding would indicate a sign of herself. What action should the therapist take?
tonsialemia? a. Arrange for voluntary hospitalization, if the
a. Seizure patient is willing
b. ECG changes b. None, because people who speak of
c. Dyspnea committing suicide seldom do it
d. Oliguria c. Arrange for immediate hospitalization
d. Request permission to speak with the
5. In planning care for a 7-year-old client with Grave’s husband to suggest marriage counseling
disease, what should the nurse do?
a. Encourage frequent rest periods 13. A young adult client is scheduled for her first
b. Encourage strenuous physical activity debridement of a second-degree burn of the left
c. Administer thyroid hormone replacement arm. It is most important for the nurse to take which
daily of the following actions?
d. Encourage a decrease caloric intake a. Assemble all necessary supplies and
medications
6. The nurse recognizes which of the following as early b. Plan adequate time for the dressing change
sign of lithium toxicity? and provide emotional support
a. Restlessness, shuffling gait, involuntary c. Prepare the client and family for the pain
muscle movements the client will experience during and after
b. Ataxia, confusion, seizures the procedure
c. Fine tremors, nausea, vomiting, diarrhea d. Limit visitation prior to the procedure to
d. Elevated white blood cell count, orthostatic reduce stress
hypotension
14. The nurse is caring for a 67 years old man following a
7. The nurse is preparing to do a shift assessment on a cardiac catheterization. Two hours after the
client who was admitted with an upper procedure, the nurse checks the patient’s insertion
gastrointestinal bleed. Which signs and symptoms site in the antecubital space, and the patient
would indicate active bleeding? (Select all that complains that his hand is numb. The nurse should:
apply) a. Change the position of his hand
a. Blood pressure 80/52 mmHg b. Check his grip strength in both hands
b. Stool black and tarry c. Notify the physician
c. Hemoglobin 18g/dL d. Instruct the patient to exercise his fingers
d. Hematocrit 32%
e. Heart rate 128 beats/min 15. A client is to receive 1000mL of IV fluid over 10
f. Respirations 32 and shallow hours. The IV tubing set calibration is 15gtt/mL.how
8. Mr. Rollins is a known alcoholic who is brought to the many drops per minute would the nurse give?
Emergency Department by the police. He has severe a. 125gtt/min
ascites from his chronic alcoholism and the physician b. 115gtt/min
prescribes spironolactone 50mg orally now. The c. 25gtt/min
pharmacy dispenses spironolactone 25mg tablets. d. 100gtt/min
How many tablets do you give Mr. Rollins?
a. 2 tablets
16. Which intervention would the nurse anticipate for a
patient who is diagnosed with osteitisdeformans 23. A 47 year old woman comes to the outpatient
(Paget’s disease)? psychiatric clinic for treatment of a fear of heights.
a. Biphosphate and recommended doses if The nurse knows that phobias include:
calcium and vitamin D a. Projection and displacement
b. Calcitonin and vitamin A supplements b. Sublimation and internalization
c. Estrogen and physical therapy c. Rationalization and intellectualization
d. A low-phosphorus and aerobic exercise d. Reaction formation and symbolization

17. The nurse has just received report from the previous 24. When teaching a client with myasthenia gravis about
shift. Which of the following clients should the nurse the management of the disease, what advice should
see first? the nurse give to the patient?
a. A client who is receiving a blood transfusion a. Prevent structured, active exercises at least
and complains of a dry mouth twice a week to prevent muscle atrophy
b. A client who is scheduled to receive heparin b. Protect extremities from injury due to
and PTT is 70 seconds decreased sensory perception
c. A client who is receiving ciprofloxacin c. Arrange a routine to accommodate frequent
(Cipro) and complains of fine macular rash visits to doctor’s office
d. A client who is receiving IV potassium and d. Perform necessary physically demanding
complains of burning at the IV site activities in the morning

18. Refer to the following list of drug indications, 25. Client with paranoid thinks he is the son of the US
actions and side effects. Which statement matches president. During interaction, he began to have
with simvastatin (Zocor)? hallucinations again, which action should be done by
a. Lowers LDL level, increase HDL level, and the nurse first?
slows progression of coronary artery a. Ignore the hallucinations and proceed with
disease. Adverse effects may include the interaction
myopathy, and hepatotoxicity b. Recognize the patient’s anxiety the proceed
b. Lowers LDL, triglycerides, and with the interaction
apolipoprotein B levels by blocking c. Let the hallucinations be the center or topic
absorption in the gastrointestinal tract. of the interaction
Minimal adverse effects have been d. Ask for help
identified.
c. Lowers LDL cholesterol and VDL triglyceride 26. Your patient has been admitted in preterm labor and
levels; raises HDL. May cause severe is receiving magnesium sulfate as a tocolytic. You
flushing prepare her for the common side effects of this
d. Reduces VLDL and increases HDL levels. medication, which include drowsiness, lethargy,
Gastrointestinal disturbances and an feeling warm and
increased risk of gallstones may occur. a. Palpitations
b. Muscular weakness
19. A client is admitted with irritable bowel syndrome. c. Tremulousness
The nurse would anticipate the client’s history to d. Tachycardia
reflect which of the following?
a. Pattern of alternating diarrhea and 27. A client has just been admitted after sustaining a
constipation second-degree thermal injury to his right arm. Which
b. Chronic diarrhea stools occurring 10-12 of the following nursing observations is most
times per day important to report to the doctor?
c. Diarrhea and vomiting with severe a. Pain around the periphery of the injury
abdominal distention b. Gastric pH less than 6.0
d. Bloody stools with increased cramping after c. Increased edema of the right arm
eating d. An elevated hematocrit

20. The nurse is preparing to do postoperative


assessment on a 5 year old child who has undergone 28. Which drug would the nurse question?
tonsillectomy. During the assessment, the nurse a. Prozac for client with bulimia
should be alert for bleeding. Which signs and b. Seroquel for patient with undifferentiated
symptoms would indicate active bleeding? (select all schizophrenia
that apply) c. Olanzapine for OC
a. Drowsiness d. Buspar for client with anxiety
b. Dark red vomitus
c. Mouth breathing
29. A patient with Raynaud’s disease should be taught to
d. Frequent swallowing
avoid which environmental factor?
e. Frequent clearing of throat
a. High levels of smog
21. The nurseis preparing to administer carvedilol b. Cold temperature
(Coreg) to a patient. Which action should the nurse c. Exposure to secondhand smoke
take first? d. Contact with pesticide
a. Find the results of the patient’s last blood
pressure measurement 30. Which statement by a 7-year-old client would
b. Check the patency of the patient’s IV line indicate an understanding of when to take
c. Assess the patient’s current pulse and blood medication (via inhaler)?
pressure a. “After one puff, I can immediately give
d. Review the patient’s urine output as myself another puff”
recorded by nurses on the previous shift.
b. “I need to depress the top of the inhaler as
22. Who to see first? I begin to take a breath.”
a. Post colectomy with abdominal cramping c. “When I remove the inhaler, I can exhale
b. Patient with post bone marrow transplant through my mouth.”
with diarrhea d. “I need to inhale the medicine and then
c. Patient with cast 30 minutes ago with hold my breath to the count of 10.”
muscle spasm
d. Patient chemo with n/v
31. To detect diabetic ketoacidosis (DKA), which of d. The nurse should model acceptable
following would you test for ketones? behavior and language for all patients.
a. Plasma
b. Feces 37. You are caring for a 7-year-old client with a brain
c. Urine tumor. Which observation would alert you to the
d. Sputum possible development of syndrome of inappropriate
antidiuretic hormone secretion (SIADH)?
32. A client has been diagnosed with metastatic cancer a. Serum sodium of 130 mEq/L
with a poor prognosis. Recently, the client has b. Weight loss
complained of increased pain and is less c. Urinary output of 30mL/h
communicative, very irritable, and anorexic. Which d. Peripheral edema
of the following nursing goals should be a priority at
this time? 38. The nurse is preparing a client for a skin biopsy.
a. Encourage client to talk about the Which of the following client statements should the
possibility of dying. nurse report to the physician?
b. Provide pain assessment and effective pain a. “I’ve been taking aspirin for my sore
management knees.”
c. Manage nutrition and hydration b. “Using lotion has helped my dry skin.”
d. Verify that the physician has discussed the c. “I went to the tanning salon yesterday.”
prognosis with the family d. “I had a big breakfast this morning.”

33. A 60-year old man with a diagnosis of pneumonia is 39. Endoscopic Retrograde Choliangopancreatography
being admitted to the medical/surgical unit. The (ERCP) SATA
nurse should place the patient in a room with which a. Check for gag reflex postprocedure
of the following patients? b. No discomfort post-procedure
a. A 20-year-old in traction for multiple c. Anesthesia will be used
fractures of the left lower leg d. No special prep needed
b. A 35-year-old with recurrent fever of e. NPO prior to procedure
unknown origin f. Will stay in hosp for a few days post op
c. A 50-year-old recovering alcoholic with
cellulitis of the right foot 40. The patient is taking ibandronate (Boniva) for the
d. An 89-year-old with Alzheimer’s disease prevention of osteoporosis. Which statement should
awaiting nursing home placement be part of the patient education provided by the
34. A patient is hospitalized for severe pregnancy- nurse?
induced hypertension (PIH). Her hematocrit has a. “Take the medication with a minimal
increased two points since the previous day. What is amount of fluid just before bedtime.”
the probable cause of this increase? b. “Take the drug first thing in the morning
a. A shift of red blood cells from the fetus with a full glass of milk or juice.”
b. A shift of fluid from the vascular c. “Take the medication on a full stomach
compartment immediately after meal.”
c. Decreased red blood cell destruction by the d. “Take the medication in the morning with a
spleen glass of water and then don’t ingest
d. Increased hematopoiesis in the red bone anything for 30 minutes.”
marrow
41. The nurse has administered sublingual nitroglycerin
35. Lucy is a 34-year-old married woman with chronic (Nitrostat) to a client complaining of chest pain.
low self-esteem. Which action by Lucy demonstrates Which of the following observations is most
assertive behavior and positive interpersonal important for the nurse to report to the next shift?
relationships? a. The client indicates the need to use the
a. Lucy requests that her husband join her bathroom.
weekly sessions to deal with the husband’s b. Blood pressure has decreased from 140/80
use of alcohol and extramarital affair. to 90/60.
b. Lucy cries for 28 minutes of the 30-minute c. Respiratory rate has increased from 16 to
therapy session 24.
c. Lucy says to the nurse, “My husband’s d. The client indicates that the chest pain has
behavior gives me headaches, so I sleep a subsided.
lot.”
d. Lucy says to the nurse, “I am going to make 42. In planning care for a client with cirrhosis who was
other people’s lives as miserable as mine admitted with bleeding esophagealvarices, to which
is.” goal should the nurse assign the highest priority?
a. Maintain fluid volume
36. The nurse on a psychiatric unit of the hospital b. Relieve client’s anxiety
refuses to agree to a 32-year-old patient’s request to c. Maintain airway patency
organize a party on the unit with his friends. The d. Control the bleeding
patient becomes angry and uses abusive language
with the nurse. Which of the following statements 43. A client is admitted to the neurology unit for a
indicates that the nurse has an understanding of the myelogram. It would be most important for the nurse
patient’s behavior? to ask which of the following questions?
a. Allowing the patient to use abusive a. “Do you have any allergies?”
language will undermine the authority of b. “Have you been drinking lots of fluids?”
the nurse. c. “Are you wearing any metal objects?”
b. Responding in kind to a patient who uses d. “Are you taking medication?”
abusive language will perpetuate the
behavior. 44. A nursing assistant is assigned to constant
c. Abusive language is one the behaviors that observation of a suicidal patient, and the nurse
is a symptom of the patient’s illness. overhears the nursing assistant talking with the
patient. Which of the following statements made by e. Weight bearing is permitted on the right
the nursing assistant would require immediate foot
intervention by the nurse? f. Weight bearing is permitted on the left foot
a. “Let’s put your clothes in the dresser.” g. The axillary area supports the body weight
b. “I’ll stay in the bathroom with you while
you take your shower.” 52. Which patient is robust?
a. Pulse pressure of 40
c. “You’re going to be moved to private room
b. BP of 90/60
later today.” c. RR of 8
d. “I’ll be right back with something for you to d. CVP of 30cmH2O
eat.”
45. A patient diagnosed with angina is instructed to rest 53. What action should the nurse take when performing
when having an episode of chest pain. What is the intermittent nasogastric (NG) feedings in a client?
best explanation for how rest relieves the pain SATA
associated with angina? a. Keep the head of the bed elevated at 15
a. Increased venous return to the heart degrees
decrease myocardial oxygen needs. b. Irrigate the NG tube prior to initiating
b. Coronary arteries constrict and shunt blood feeding
to vital areas of the myocardium. c. Deliver feedings through a syringe barrel
c. A balance between myocardial cellular attached to the NG tube
d. Deliver the feeding by pushing on the
needs and demand is achieved.
syringe plunger
d. Coronary blood vessels dilate and increase e. Aspirate the stomach contents
myocardial cell perfusion. f. Clamp the NG tube once the feeding is
complete
46. Twelve hours after a total thyroidectomy, the client
develops stridor on exhalation. What is the nurse’s
54. A patient with chronic mental health problems has
best first action?
been making progress with treatment. During the
a. Hyperextend the client's neck.
most recent visit to the clinic, however, the patient
b. Reassure the client that the voice change is
tells the nurse he lost his job and feels useless
temporary.
because he is unable to provide for the family.
c. Call for emergency assistance.
Which nursing diagnosis would be most appropriate
d. Document the finding as the only action.
for this patient?
a. Social isolation
47. Which of the following is the first nursing action that
b. Caregiver role restrain
should be implemented for a 25-year-old woman
c. Situational low self-esteem
after a vaginal delivery?
d. Anxiety
a. Check the patient’s lochial flow
b. Palpate the patient’s fundus
55. To minimize the side effects of a DPT immunization
c. Monitor the patient’s pain
for a six-month-old, the nurse should instruct the
d. Assess the patient’s level of consciousness
parents to:
a. Give the child an alcohol bath for an
48. The nurse is caring for a client receiving
elevated temperature
amphotericin B (Fungizone) 1mg in 250cc of 5%
b. Administer antipyretics for discomfort,
dextrose in water IV over a 2-hour period. The nurse
irritability, and fever
should be most concerned if which of the following
c. Place an ice bag on the child’s leg for three
was observed?
days
a. BUN 7.2 mg/dL, creatinine 0.5 mg/dL.
d. Check the child’s temperature every four
b. BP 90/60, complaints of fever and chills.
hours for three days
c. Complaints of burning on urination, thirst,
and dizziness.
56. On admission, the vital signs of a client with a closed
d. AST (SGOT) 12 U/L, ALT (SGPT) 14 U/L,
head injury were temperature of 98.6F, blood
total bilirubin 0.2 mg/dL.
pressure 128/68mmHg, heart rate 110beats/min,
respiration 26. One hour after admission, the nurse
49. What equipment would be necessary for the nurse to
observes that the client may be experiencing
complete an evaluation of cranial nerve III during a
Cushing’s triad. Which vital signs are indicative of
physical assessment?
Cushing’s triad?
a. Tongue depressor
a. Blood pressure 110/70mmHg, heart rate
b. A pen light
120beats/min, respiration 30
c. A cotton swab
b. Blood pressure 130/72mmHg, heart rate
d. A safety pin
90beats/min, respiration 24
c. Blood pressure 152/88mmHg, heart rate
50. A G1P0 30-year-old patient at 38 weeks gestation is
122beats/min, respiration 16
admitted with heavy, bright red bleeding. The initial
d. Blood pressure 150/70mmHg, heart rate
nursing assessment should include all of the
80beats/min, respiration 14
following except?
a. Fetal monitoring
57. A female client is diagnosed with human
b. Asking about the pain
papillomavirus (HPV). Which of the following client
c. Taking vital signs
statements, if made to the nurse, illustrates an
d. A vaginal examination
understanding of the possible sequelae of this
illness?
51. A 12-year-old client has a right tibia fracture that is
a. “I will need to take antibiotics for at least a
casted. The client needs instruction regarding how
week.”
to walk in crutches using a three-point gait prior to
b. “I will use only prescribed douches to avoid
be discharged from the Emergency Department.
a recurrence.”
Which instructions would be included? (SATA)
c. “I will return for a Pap smear in six
a. The hands and arms support the body’s
months.”
weight
d. “I will avoid using tampons for eight weeks.
b. The body swings through and beyond the
crutches
58. The nurse is caring for a client with a cervical spinal
c. The right foot acts like a balance
cord injury. Vital signs and laboratory results for this
d. Advance both crutches and swing both feet
client are as follows:
forward
Blood pressure: 128/72 mmHg a. From 28-52 weeks
Heart rate: 94 beats/min b. Up to 3 minutes
Arterial pH: 7.3 c. Up to 30 minutes
Arterial pCO2: 60 mmHg d. From 2 to 8 weeks
Arterial pO2:75 mmHg
Arterial HCO3: 35 mEq/L 65. A patient who is 28 weeks pregnant complains of
Based of this information which nursing action would lower back pain. What should the nurse suggest?
be the best action? a. The patient take Motrin as needed
a. Notify the physician, request an order for b. Lower back pain is part of being pregnant
midazolam, and reevaluate the client in 30 and there is nothing the patient can do
minutes about the pain
b. Evaluate airway patency, place the client in c. The patient pay close attention to her body
high Fowler’s position, and encourage posture and mechanics, as these are the
coughing and deep breathing cause of back pain in pregnancy
c. Notify the physician, inform the physician d. The patient tell her provider immediately,
about the client’s metabolic acidosis and because she is in preterm labor
anticipate a sodium bicarbonate continuous
infusion 66. The nurse has collected the following data: client
d. Evaluate airway patency, administer pain anger directed toward staff in the form of frequent
medication and encourage coughing and sarcastic or crude comments, increased wringing of
deep breathing hands, and purposeless pacing, particularly after the
client has used the telephone. Based on this data,
59. A patient diagnosed with gout asks, “Is there the nurse should make which nursing diagnosis?
anything I can do to decrease my uric acid levels?” a. Impaired social interaction related to
What is the nurse’s most appropriate response? conversion reaction
a. Avoid strenuous activity, as it will cause b. Risk for potential activity intolerance as
muscle breakdown. evidenced by purposeless pacing
b. Decrease the amount of liver, sardines, and c. Powerlessness in hospital situation
shrimp in your diet d. Ineffective individual coping related to
c. Increase the amount of citrus fruits in your recent anger and anxiety
diet
d. Drink at least 1 to 1.5 liters of fluid each 67. An adult patient’s prescription reads as follows, “
day. Infuse 80 mEq of potassium chloride in 100 cc D5W
over 30 minutes. Based on the nurse’s understanding
60. During the nursing history interview, a preschool of potassium administration, what is the most
client’s mother reports that the child has frequent appropriate action?
bouts of gastroenteritis. It would be most important a. Contact the prescriber about the order
for the nurse to ask which of the following questions? b. Monitor the EKG during the medication’s
a. “Are there other children in the family?” administration
b. “Does the child attend a day care center?” c. Switch the administration route to oral
c. “Does the child play with neighborhood d. Administer the medication
children?”
d. “Is the child current on his immunizations?” 68. The nurse’s aide comes to take a woman by
wheelchair for a magnetic resonance imaging (MRI)
61. A 9-year-old client is given his heparin injection on scan of the head and neck. Which of the following
time, but it was administered intravenously instead observations, if made by the nurse, would require an
of subcutaneously. The incident was discovered 2 intervention?
hours after administration. Which plan would be a. The woman removes her dentures and gives
most appropriate? them to her husband.
a. Document the event on an incident report b. The woman’s vital signs are: BP 120/70,
and notify the physician pulse 80, respirations 12, temperature 99°F
b. Hold the next scheduled heparin dose (37.3°C).
c. Order a PTT and INR levels and notify the c. The woman has a nitroglycerine patch on
physican her right chest area.
d. Assess for evidence of bleeding and notify d. The woman has red nail polish on her
the parents fingers and toes.
69. A middle-aged man is admitted to an inpatient
62. A client is diagnosed with lung cancer and undergoes psychiatric unit. Over the last several months he has
a pneumonectomy. In the immediate postoperative become convinced that his brother is trying to steal
period, which of the following nursing assessment is his property. He is diagnosed with paranoid disorder.
most important? The nurse knows that this client is demonstrating
a. Presence of breath sounds bilaterally. which of the following
b. Position of the trachea in the sternal notch. a. Delusions of persecution
c. Amount and consistency of sputum. b. Command hallucination
d. Increase in the pulse pressure. c. Delusions of reference
d. Persecution hallucination
63. The nurse receives a phone call from a nursing
assistant who states that her five-year-old child has 70. A client returns to the unit from the recovery room
developed chickenpox. It would be most important following a laryngoscopy. Which position would be
for the nurse to ask which of the following questions? most effective in helping the client breathe?
a. “Have your other children had chickenpox?” a. Side-lying position
b. “Does your child have a temperature?” b. Sims position
c. “Have you had the chickenpox?” c. Low fowler’s position
d. “Do you have someone to watch your d. Trendelenburg position
child?”
71. For the following herbal supplement, select the
64. Mr. Holloway has just received his first dose of this purported use of: Black cohosh (Cimcifugaracemosa)
antipsychotic medication perphenazine (Trilafon) a. Used to relieve symptoms associated with
you know that the response time to the medication benign prostatic hypertrophy
for cognitive and perceptive symptoms, such as b. Used to relieve symptoms of menopause
hallucinations, delusions and thought broadcasting, c. Used to relieve depression
may take how long?
d. Used to improve memory, sharpen d. A client with congestive heart failure who
concentration and promote clear thinking underwent diuresis in the hospital

72. What is the highest priority in providing care to a 79. Which instruction would be given to a client who is
client who is admitted to the hospital with sickle cell receiving oral methylprednisolone regarding when
crisis? and how to take the medication?
a. Insist the client rest instead of visiting with a. Once a day before bedtime
family b. Consume 10-12 glasses of water per day
b. Administer prophylactic antibiotics c. Once a day on an empty stomach
c. Initiate intravenous fluids to maximize d. Once a day with breakfast
hydration
d. Insert urinary catheter to measure accurate 80. A patient with osteoarthritis has had hip
output replacement surgery. What level of activity would
the nurse anticipate for the first postoperative day?
73. To maintain normalized blood sugars, Mr. Hernandez a. Paresthesia, rigidity, aphasia
has the following sliding scale insulin prescription: b. Tremors, rigidity, bradykinesia
Blood glucose < 130mg/dl: administer 0 unit of c. Spasticity, diplopia, paresthesia
insulin d. Dysarthria, dysphagia, ataxia
Blood glucose 130-160mg/dl: administer 2 unit of
insulin 81. A patient with acute coronary syndrome is
Blood glucose 161-190mg/dl: administer 4 unit of administered thrombolytic therapy. Which portion of
insulin the EKG tracing would the nurse observe to
Blood glucose 191-220mg/dl: administer 6 unit of determine the effectiveness of the medication?
insulin a. ST segment elevation
Blood glucose 221-250mg/dl: administer 8 unit of b. PR interval
insulin c. QT interval
Blood glucose >250mg/dl: administer 10 unit of d. Width of QRS complex
insulin and contact the physician immediately
Mr. Hernandez blood sugar is 122. What is your 82. The nursing assistant reports to the nurse that a
intervention? client who is one-day postoperative after an
a. administer 2 unit of insulin angioplasty is refusing to eat and states, “I just don’
b. administer 4 unit of insulin t feel good.” Which of the following actions, if taken
c. administer no of insulin and contact the by the nurse is best?
physician immediately a. The nurse talks with the client about how
d. administer no insulin he is feeling
b. The nurse instructs the nursing assistant to
74. A patient with Alzheimer’s disease doesn’t want to sit with the client while he eats
take a bath, what will the nurse do? c. The nurse contacts the physician to obtain
a. Call two staff nurses to help you bathe the an order for an antacid
patient d. The nurse evaluates the most recent vital
b. Attempt to bathe the patient slowly and signs recorded in the chart
calmly
c. Ask the patient the reason why she doesn’t 83. An 18 month old client is admitted to the hospital
want to take a bath with a fever of 104F, respirations of 56/min,
d. Document refuse to take a bath suprasternal retractions and a pulse oximeter
reading of 85%; the infant is also drooling. Acute
75. Mr. Allen has psychosis and has been treated with epiglottitis is suspected. Which equipment would be
haloperidol (Haldol). You need to assess him for important to have at the bedside?
movement disorders as a side effect of Haldol. What a. Tracheostomy tray
is another name for these movement disorders? b. Intravenous infusion pump
a. Delusion etiologies c. Defibrillator
b. Extrapyramidal reactions d. Code cart
c. Autonomic dysreflexia
d. Biologic rigidity reactions 84. Match the eating disorder with the correct
description of the disease. An excessive concern over
76. Identify the location on the chest area where the gaining weight and a refusal to maintain a minimally
nurse would take an apical pulse. normal body weight.
a. Right 5th intercostal space, midclavicular a. Bulimia nervosa
line b. Anorexia nervosa
b. Left 8th intercostal space
c. Right 8th intercostal space 85. Among the four patients, who warrants immediate
d. Left 3rd intercostal space, midclavicular line attention?
a. Patient taking Glucophage with glucose
77. A client with deep vein thrombophlebitis suddenly reading of 185mg/dl
develops dyspnea, tachypnea, and chest pain. What b. Patient who had a spinal injury and is
is the nurse’s initial, most appropriate action? complaining of throbbing headache
a. Apply 100% oxygen via face mask c. Patient diagnosed with seizure who wants
b. Obtain a 12-lead ECG to change medication time
c. Assess the client’s blood pressure and heart d. Patient with osteoarthritis experiencing
rate joint stiffness
d. Auscultate for abnormal heart sounds
86. Which drug would the nurse anticipate administering
78. The nurse is planning discharge for a group of for the treatment of inflammation of acute
clients. It is most important to refer which of the exacerbations of gout?
following clients for home care? a. Allopurinol (Zyloprim)
a. A postoperative appendectomy client who is b. Acetaminophen (Tylenol)
complaining of incisional pain c. Probenecid (Benemid)
b. A diabetic client who had a cardiac d. Colchicine (novocholchicine)
catheterization in the early AM
c. A postoperative cholecystectomy client who 87. A client returns to the unit undergoing a right
is complaining of incisional pain modified radical mastectomy with dissection of the
axillary lymph nodes. Which measure is an
appropriate intervention for the nurse to include in 94. A 4-year-old has been admitted with second-degree
the client’s postoperative care? burns and is undergoing debridement of the wounds.
a. Encourage the client to obtain a permanent Morphine 1 mg IV push has been administered.
breast prosthesis upon discharge from the Following administration of this medication, the
hospital nurse makes the following observations:
b. Instruct the client to watch the clock and Pulse: 96
use the PCA pump every 10 minutes Respirations: 28
c. Insist that the client examine the surgical Blood pressure: 84/62
incision when the surgical dressings are Child sleeping quietly
removed Which nursing action is most appropriate?
d. Post a sign at the bedside to avoid pressure a. Keep the code cart at the bedside
measurements or venipunctures in the right b. Allow the child to sleep quietly
arm. c. Administer nalozone (Narcan)
d. Administer 100% oxygen
88. Patient with HIV states, “I am so tired, my life is
useless, I am going to die anyway.” 95. Individuals with diabetes mellitus can have a chronic
a. Refer to support group complication in which there is pain in the lower
b. Why are you tired? extremities due to lack of blood supply. What is the
c. What is the specific cause that makes you complication called?
feel tired? a. Retinopathy
d. Do you think you are dying? b. Claudication
c. Stroke
89. After receiving a total hip replacement, the client d. Angina
returns to the unit with an abductor pillow in place.
The client informs the nurse that he would be more 96. The nurse is assessing an infant who had a repair of a
comfortable without the pillow. What is the nurse’s cleft lip and palate. The respiratory assessment
best response? reveals that the infant has upper airway congestion
a. “The pillow is intended to prevent the and slightly labored respirations. Which of the
inadvertent movement of the left leg following nursing actions would be most appropriate?
beyond the body’s midline.” a. Elevate head of the bed
b. “The pillow is intended to prevent early b. Suction the infant’s mouth and nose
ambulation if you should wake up c. Position the infant on one side
confused.” d. Administer oxygen until breathing is easier
c. “The pillow is intended to prevent the
inadvertent movement of the left leg too 97. Why is maintaining a thermoneutral environment
far way from the body.” essential for the neonate?
d. “The pillow is intended to prevent the a. A thermoneutral environment permits the
contact of both knees and reduce the risk neonate to maintain a normal core
that pressure ulcers will form.” temperature with increased caloric
consumption
90. The triage nurse for a women’s health center b. Metabolism slows dramatically in the
receives a phone call from each of the following neonate who experiences cold stress
women. Which woman should be directed to come to c. The neonate produces heat by increasing
the health care facility immediately? activity and shivering
a. A multipara woman who is four weeks d. A thermoneutral environment permits the
pregnant and reporting unilateral, dull, neonate to maintain a normal core
abdominal pain. temperature with minimum oxygen
b. A primipara woman who is seven weeks consumption
pregnant and reporting increase in whitish
vaginal secretions. 98. A six-month-old infant has had all the required
c. A primigravida woman who is five weeks immunizations. The nurse knows this would include
gestation and is having vaginal spotting and which of the following?
some cramping. a. Two doses of diphtheria, tetanus, and
d. A multigravida woman who is six weeks pertussis vaccine
pregnant and reporting frank, red vaginal b. Measles, mumps and rubella vaccines
bleeding with moderate cramps. c. A booster dose of trivalent oral polio
vaccine
91. Erythromycin ophthalmic ointment 0.5% is given d. Chickenpox and smallpox vaccines
immediately after an infant is born to provide
prophylaxis against: 99. A 4-year-old child presents with possible rheumatic
a. Chlamydia trachomatis fever. Which findings will the nurse observe in this
b. Syphilis patient?
c. Both Neisseria gonorrhoea and chlamydia a. Decreased antistreptolysin-O titer
trachomatis b. Decreased erythrocyte sedimentation rate
d. Neisseria gonorrhoea c. Macular rash that is pruritic
d. Elevated C-reactive protein levels
92. Which food choice would be most appropriate for a
patient with osteoporosis who wants to increase 100.The nursing evaluation of the respiratory status of a
calcium intake? 3-year-old client who is newly admitted with acute
a. 1 ounce of cream cheese epiglottitis would indicate the following findings:
b. 1 medium stalked of cooked broccoli a. Drooling, decreased pulse and stridor
c. 3 ounces of beef b. Irritability, drooling and absence of
d. 1 medium apple spontaneous cough
c. Irritability, coarse crackles bilaterally and
93. Which EKG tracing would the nurse recognize as an low-grade fever
early indicator of hyperkalemia? d. Croupy cough, high fever and hoarseness
a. Depressed ST segment
b. Prolonged QT interval 101.A client with necrotizing spider bite is to perform his
c. Shallow, flat, inverted T wave own dressing changes at homes. The nurse is aware
d. Tall peaked T-wave that which of the following statements, if made by
the client, indicates understanding of aseptic
technique?
a. “I need to buy sterile gloves to redress this c. Auscultate lung sounds and suction if
wound.” needed
b. “I should wash my hands before redressing d. Notify the physician and prepare for
my wound.” immediate surgery
c. “I should keep the wound covered at all
times.” 108.A 22-month-old client is receiving Nystatin 200,00
d. “I should use an over-the-counter units via oral swab every 6 hours. For which side
antimicrobial ointment.” effects should the nurse assess the client?
a. Leukopenia
102.A 22-month-old child is hospitalized for heart failure. b. Oral thrush
During the night, the child awakens crying and c. Diarrhea
calling for the mother. The nurse assesse the child d. Thrombocytopenia
and notes dyspnea, jugular vein distention, crackles
and pink, frothy sputum. After the nurse begins 109.The nurse is performing discharge teaching on a
oxygen by 40% face mask, which action should be client with multiple sclerosis. It is most important
taken next? for the nurse to include which of the following
a. Dim the lights and allow the mother to rock instructions?
the child to sleep a. Ambulate as tolerated every day
b. Continue to monitor the client frequently b. Avoid overexposure to heat or cold
and increase fluid rate c. Perform stretching and strengthening
c. Place the child in a crib with a blanket and exercises
notify the physician d. Participate in social activities
d. Stay with the child and call for assistance to
notify the physician 110.A client is being prepared for surgical repair of an
abdominal aortic aneurysm. The nurse suspects
103.An elderly patient has been prescribed aspirin for complete aortic dissection when:
osteoarthritis. What should the nurse teach the a. The client becomes hypotensive and
patient to ensure safe use of this medication? unresponsive
a. A prothrombin time should be drawn upon b. A bruit and thrill are palpable at the
initiation of therapy and every 2 months aneurysm site
b. The prescriber should be notified if the c. The client becomes hypertensive and
patient experiences any unusual bruising or tachycardic
bleeding d. The client complains of sever leg and arm
c. The medication should be taken on an pain
empty stomach
d. Enteric-coated tablets should be crushed to 111.The nurse is caring for patients on the surgical floor
make the medication easier to swallow and has just received report from the previous shift.
Which of the following patients should the nurse see
104.The nurse is caring for client in the outpatient clinic. first?
Which of the following messages should the nurse a. A 35-year-old admitted 3 hours ago with a
return first? gunshot wound, 1/5 cm area of dark
a. A mother reports that the umbilical cord of drainage noted on the dressing.
her five-day-old infant is dry and hard to b. A 43-year-old who had mastectomy two
touch. days ago, 23 cc of serosanguinous fluid
b. A mother reports that the “soft spot” on noted in the Jackson-Pratt drain
the head of her four-day-old infant feels c. A 59-year-old with a collapsed lung due to
slightly elevated when the baby sleeps. an accident, no drainage noted from the
c. A mother reports that the circumcision of chest tube in the last eight hours
her 3-day-old infant is covered with d. A 62-year-old who had an abdominal-
yellowish exudate. perineal resection three days ago, patient
d. A father reports that her bumped the crib complains of chills
of his two-day-old infant and she violently
extended her extremities and returned to 112.Which of the following statements, if made by the
their previous position. parents of a nine-year-old client with an ostomy,
would indicate to the nurse that they are providing
105.Indomethacin is given as a treatment for preterm quality home care?
labor. What is a potentially significant fetal side a. “We change the bag at least once a week
effect of this drug? and we carefully inspect the stoma at that
a. Premature closure of the ductusvenosus time.”
b. Bradycardia b. “We change the bag every day so that we
c. Decreased fetal movement can inspect the stoma and the skin.”
d. Premature closure of the ductusarteriosus c. “We encourage our daughter to watch TV
while we change her ostomy bag.”
106.The nurse is caring for a client with internal d. “We only change the ostomy bag every ten
radiation. Which of the following actions, if taken by days.”
the nurse, is most important?
a. Restrict visitor who may have an upper 113.Nursing care in the first 30 minutes after a caesarean
respiratory infection section includes:
b. Assign only male caregivers to the client a. Fundal and lochial assessment
c. Plan nursing activities to decrease nurse b. Ambulation
exposure c. Vital signs every hour
d. Wear a lead-lined apron whenever d. Oral hydration and nutrition
delivering client care
114.Education about health promotion is often effective
107.The nurse is caring for a client who sustained severe during periods of role transitions. Which of the
burns and has an inhalation thermal injury. The following is a role transition?
client is intubated and on the ventilator at 60% FiO 2. a. Retirement
The nurse notices that the client is restless, b. Buying a new car
thrashing, and attempting to cough, the respiratory c. Moving into a new house in the same
rate is 34. What should the nurse’s first action be? neighbourhood
a. Administer pain medication d. Going grocery shopping
b. Increase the FiO2 setting to 100%
115.An extremely angry patient with bipolar illness tells c. Auditory or tactile materials
the nurse he just learned his wife filed for divorce, d. Use of a slow, deliberate speech pattern
and he needs to use the phone. Which of the
following responses by the most nurse is most 123.A 19-year-old patient has just been admitted to the
appropriate? detoxification unit after drinking a quart of vodka
a. Allow the patient to use the phone every day for the past 3 weeks. What is the most
b. Confront the patient about his anger and important nursing intervention on the day of
inappropriate plan of action admission to reduce the risk of harm to this patient?
c. Do not allow the patient to use the phone a. Give the patient a meeting schedule for
because he is an involuntary patient Alcoholic Anonymous
d. Set limits on the patient’s phone use b. Administer Librium as prescribed
because he has been unable to control his c. Encourage the patient to attend group
behaviour therapy sessions
d. Explain the addictive process to the patient
116.A patient’s medicinal history includes the use of the
herbal medication garlic and the prescribed 124.The physician informs the nurse that a client needs
medication warfarin (Coumadin). Based in the nurse’ to be intubated. In preparing for the physician to
s knowledge of drug-drug interactions, which perform the intubation, which equipment is
problem could occur when a patient takes both of appropriate for this procedure? (SATA)
these products concurrently? a. Laryngoscope
a. Elevated blood pressure b. Sterile gloves
b. Decreased immune function c. Uncuffed endotracheal tube
c. Altered renal perfusion d. Oral suction
d. Increased bleeding potential e. Face mask
f. Ambu bag
117.Which statement by a patient would indicate that
the patient is adapting well to changes in functional 125.A 23-year-old man comes to the AIDS clinic for
status after experiencing a spinal cord injury? treatment of large, painful, purplish-brown open
a. “I tire easily when I use my wheelchair just areas on his right arm and back. The nurse should
around the house. I know I would get tired instruct the client to:
if I tried to leave the house.” a. Clean the area carefully with soap and
b. "My wife tries to get me to go to the grocery warm water every day and cover them with
store, but I don't like to go out much." sterile dressing
c. "I have all the equipment to take a shower, b. Soak in a warm tub twice a day and rub the
but I prefer a bed bath, because it is areas with a washcloth before covering
easier." them
d. "I have been using the modified feeding c. Shower daily using a mild antimicrobial soap
utensils at every meal. I still have spills, but from a pump dispenser and leave the lesions
I'm getting better." uncovered
d. Clean the lesion twice a day with a diluted
118.A client is receiving plasmapheresis treatments for solution of povidone-iodine (Betadine) and
myasthenia gravis. Which observation would the leave them open to the air
nurse identify as the desired response for this
treatment? 126.The nurse is admitting a client to the unit from the
a. Increased ptosis postoperative recovery area after abdominal
b. Decreased functional residual capacity exploratory surgery. After determining the client’s
c. Ability to consume an entire meal vital signs, which of the following activities should
d. Need for frequent rest periods the nurse perform next?
a. Position the client on her left side,
119.A client is brought to the emergency room after a supported with pillows
motor vehicle accident that resulted in the client b. Check the chart and determine the status of
sustaining a head injury. Which assessment should the fluid balance from surgery
the nurse perform immediately? c. Check the client’s abdominal dressing for
a. Assessment of the respiratory status any evidence of bleeding
b. Assessment of pupils d. Monitor the incision and pulmonary status
c. Assessment of short-term memory for the presence of infection
d. Assessment of motor function
127.A physician has written an order for an HIV-positive
120.To promote safety in the environment of a client infant to receive an oral polio immunization. Which
with a marked depression of T cells, the nurse of the following nursing actions is most appropriate?
should: a. Wear gloves and gown when administering
a. Keep a linen hamper immediately outside the immunization
the room b. Administer the immunization as infant is
b. Restrict eating utensils to spoons made of being discharged
plastic c. Call the physician and discuss the rationale
c. Provide masks for anyone entering the room for the immunization
d. Remove any standing water left in d. Administer the medication in the same
containers or equipment manner as you would to any other infant

121.A nurse is caring for a client with a spinal cord 128.Mrs.Tungen, who has been diagnosed with bipolar
injury. Which observation would indicate this client disorder, is receiving lithium and outpatient
is exhibiting neurogenic shock? therapy.She now complains of diarrhea, vomiting,
a. Heart rate of 52 beats/min thirst, and coarsening hand tremors. What should the
b. Temperature of 102.5F nurse's first intervention be?
c. Heart rate of 115 beats/min a. Hold the lithium, and call for an order to
d. Cool, moist skin obtain a lithium level.
b. Administer an antidiarrheal medication.
122.The nurse is preparing a teaching plan for a patient c. Obtain a stool sample for culture.
who is visually impaired. Which teaching strategy d. Begin an intravenous drip of D5 ½ NS with
should be included in the plan? 20 mg potassium chloride to infuse at 125
a. Provision of written information mL/h.
b. Use of captioned video materials
129.The nurse observes a client who is taking phenelzine Mucinex 600-mg extended-release tablets. How
(Nardil) eating another client’s lunch. After a few many tablets doo you give Mr. Lawrence?
minutes, the client complains of headache, nausea, a. 2 tablets
rapid heartbeat, and begins to vomit. The nurse b. None
anticipates administering which of the following c. 0.5 tablet
medication? d. 1 tablet. None.
a. Buspirone (Buspar)
b. Fluoxetine (Prozac) 137.On the second day after a subtotal thyroidectomy,
c. Prochlorperazine (Compazine) the client informs the nurse that she is experiencing
d. Nifedipine (Procardia) numbness and tingling around her mouth. What is
the nurse’s best first action?
130.An infant born with spina bifida and is scheduled for a. Notify the physician
surgery the next day. Which nursing action has the b. Order a thyroid-stimulating hormone level
greatest priority? c. Loosen the neck dressing
a. Preventing infection by supine positioning d. Offer mouth care
b. Encouraging the parents to hold, cuddle and
feed the infant 138.Select all self-care activities that persons should be
c. Promoting range-if-motion exercises able to perform prior to discharge to home. (SATA)
d. Preventing rupture of the meningocele sac 1. Bathing
2. Banking
131.The nurse is caring for a client admitted with acute 3. Dressing
hypoparathyroidism. It is most important for the 4. Car oil change
nurse to have which of the following item available? 5. Grooming
a. Tracheostomy set 6. Hygiene
b. Cardiac monitor
c. IV monitor 139.When to know if a 6 year old child has a
d. Heating pad dysfunctional grieving after the death of a
grandparent?
132.Which assessment finding indicates effective chest a. The child refuses to eat and stays in his
compressions during CPR? room
a. Pink mucous membranes b. The child re-enacts the funeral using his
b. Palpable carotid pulse stuff toys and pets
c. Dilated pupils bilaterally c. The child asks when he can play with
d. Sluggish capillary refill grandpa just after visiting his grave
d. The child states that his grandpa is just
133.A nasogastric tube is ordered to be placed in a sleeping and would wake up soon
client. Organize the following steps in chronological
order as they relate to this procedure: 140.Mr. Koo is prescribed chlorpromazine (Thorazine) as
1. Lubricate the tube an antipsychotic medication. When he comes to the
2. Measure the tube for approximate placement pill line in the hospital, he reports that he has taken
length 2 days worth of the medication as prescribed and is
3. Place the client in a high Fowler’s position now experiencing dizziness. What should your first
4. Advance the tube downward and backward nursing intervention be?
5. Insert the tube along the base of the nose a. Obtain a complete blood count and serum
6. Check the position of the tube, and secure the ammonia level as prescribed
tube b. Assess blood pressure with the patient In
a. 3,1,2,5,4,6 both the lying and standing positions
b. 2,3,1,5,4,6 c. Assess the optic chiasm using an
c. 1,3,2,5,4,6 ophthalmoscope
d. 3,2,1,5,4,6 d. Obtain pulmonary function test, stat

134.A client has been receiving chlorpromazine 141.A 9-year-old client is receiving one unit of packed
hydrochloride (Thorazine). When the nurse checks on red blood cells. Which finding on assessment would
the patient, the patient is restless, unable to sit, indicate a possible haemolytic reaction? (SATA)
and complains of insomnia and fine tremors of the a. Tightness in chest
hands. The nurse identifies which of the following as b. Red or black urine
the best explanation about why these symptoms are c. Shaking
occurring? d. Temperature of 97.6F
a. A side effect of the medication that will e. Flank pain
disappear as time passes. f. Bilateral crackles
b. The reason the patient is receiving this
medication. 142.The nurse is supervising the staff caring for clients
c. Extrapyramidal side effects resulting from on the medical/surgical unit. The nurse observes the
this medication. student nurse enter wearing a gown, gloves, and a
d. An indication that the dosage of the mask. The nurse determines that the precautions are
medication needs to be increased. correct if the student nurse is caring for which of the
following clients?
135.A client has a right-side pneumothorax and a chest a. An infant diagnosed with respiratory
tube has been inserted. Which finding would indicate syncytial virus
that the chest drainage system is functioning b. A young child with a wound infected with S.
effectively? aureus
a. Blood leaking around the chest tube c. A teenager diagnosed with toxic shock
insertion site syndrome
b. Constant bubbling in the water seal d. A teenager diagnosed with rubella (German
chamber measles)
c. Absence of breath sounds on the right side
d. Bubbling in the suction chamber 143.A client is admitted in sickle cell crisis and is
receiving IV morphine by PCA pump. The nurse
136.Your patient, Mr. Lawrence, has been prescribed makes the following observations:
Mucinex (guaifenesin) 300 mg orally daily as part of Pulse: 73
his treatment for bronchitis. The pharmacy sends up Respirations: 6
Blood pressure: 112/72 mmHg
Client is quietly sleeping recommendations, if made by nurse, will be most
Which nursing action is most appropriate? therapeutic to achieve this goal?
a. Allow the client to sleep quietly a. Take a day trip with a friend
b. Administer 100% oxygen b. Take an eleven-minute bus ride alone
c. Administer naloxone (Narcan) as prescribed c. Join a support group and participate in a
d. Keep the code cart at the bedside victim assistance organization
d. Take a ten-minute with his wife around the
144.An autoimmune disorder attacks the myelin sheaths block
of nerve fibers in the central nervous system and
produces lesions called plaques. This statement 152.The client is at risk for bleeding related to the
describes the pathophysiology of which disease? Vitamin K deficiency and the altered liver functions
a. Amyotrophic lateral sclerosis a. Roasted chicken breast, baked potato with
b. Multiple sclerosis margarine and chives and skim milk
c. Alzheimer’s disease b. Two eggs, two slices of toast with
d. Myasthenia gravis margarine and a glass of whole milk
c. Baked fish, steamed broccoli with salt and
145.The multipdisciplinary team decides to implement pepper, and a glass of iced tea
behaviour modification with a client. Which of the d. Grilled cheese sandwich, steamed
following nursing actions is of primary importance vegetables with butter and a cup of coffee
during this time?
a. Confirm that all staff members understand 153.During a home visit, the nurse saw an old woman
and comply with the treatment plan. filthy and unkempt in her child’s house. What should
b. Establish mutually agreed upon, realistic you do?
goals. a. Advise the woman to visit the hospital
c. Ensure that the potent reinforcers (rewards) b. Talk to the child of the old woman
are important to the client. c. Call the abuse center for the elderly
d. Establish a fixed interval schedule for d. Perform hygiene care for the old woman
reinforcement. 154.A patient has a subcutaneous terbutaline (Brethine)
pump for treatment of preterm labor. Which of the
146.When completing discharge teaching for a patient following findings warrants a call to the physician?
who has experienced a myocardial infarction, the a. Fetal movements are fewer than 12 per
patient asks the nurse why aspirin has been hour
prescribed daily. What is the nurse’s best response? b. The patient feels nervous and jittery
a. “The medication helps to maintain coronary c. The patient’s pulse is 124 beats per minute
blood flow by decreasing platelet d. Fetal movements are more than 12 per hour
aggregation in the coronary arteries.”
b. “Aspirin is used to prevent fever associated 155.The nurse is teaching a client how to perform self-
with the inflammatory response in monitoring blood glucose (SMBG) using a blood
myocardial infarction.” glucose monitor. Which of the following actions, if
c. “The medication increases the amount of performed by the client indicates to the nurse the
blood in the coronary arteries.” need for further teaching?
d. “Aspirin is used as prophylactic analgesic to a. The client lets her hand dangle before
reduce pain.” sticking her finger with the lancet
b. The client sticks her finger on the side of
147.Which immunization should be withheld if patient the distal pharynx
experiences seizures? c. The client touches the strip with a large
a. Hepatitis drop of blood from her fingertip
b. DPT d. The client milks her finger after sticking it
c. OPV
d. Measles 156.A bipolar patient refuses to put down the mop that
he is swinging to threaten other patients and staff.
148.A 2-month-old infant is 2 days postoperative What information is most important for the nurse to
tracheoesophageal fistula repair. A complete blood consider before administering a PRN IM dose of
count reveals a haemoglobin of 8.6mg/dl and lorazepam (Ativan)?
erythrocyte count of 2.5 million/mm 3. Which a. The patient is harmful to himself
symptoms would the nurse most likely find on b. The patient is psychotic
assessment? c. A restrictive intervention failed
a. Projectile vomiting after oral bottle feeding d. The patient is harmful to others
b. Sluggish capillary refill and hypotension
c. Slight pallor and tires easily while crying 157.A client is admitted to the burn unit with a third-
d. Tachycardia and flushing degree burn to the chest, face, and upper
extremities. During the acute phase (i.e., first 48
149.The nurse is caring for a patient with acute coronary hours) of a major burn injury, which assessment
syndrome who is receiving altaplase (tPA). Which findings should the nurse report immediately?
side effects should the nurse monitor the patient a. Temperature of 100F
for? b. Edema of hands
a. Hepatomegaly c. Decreased sensation in the extremities
b. Fluid retention d. Urinary output of 200 ml over 8 hours
c. Bleeding
d. Muscle pain 158.A 19-month old child weighs 22 pounds and has an
order of 200 mcg digoxin to be given intravenously.
150.Lamotrigine is given to clients to relieve them from You have a vial of digoxin at a concentration of
what? 0.1mg/ml. how many millilitres of the solution will
a. Seizure you need to deliver the ordered dose?
b. Joint stiffness a. 0.22 mL
c. Blood pressure b. 0.002 mL
d. Migraine c. 2.0 mL
d. 0.2mL
151.One of the goals the nurse and a client with
posttraumatic stress disorder (PTSD) mutually agreed 159.Mrs. Langley has hyperosmolar nonketotic coma with
upon is that he will increase his participation in out- hyperglycemia. She begins to experience CNS
of-the apartment activities. Which of the following
dysfunction. What is most likely source of this b. Tell the patient that he may experience
dysfunction? discomfort because of the needles to be
a. Adrenal gland tumor used
b. Cellular fluid loss c. Put the patient on NPO 6-8 hours
c. Fever d. Ask the patient to empty the bladder
d. Hypoxia
167.Indicated use of magnesium sulphate in pregnancy is:
160.The nurse is supervising a care given to clients on a a. To prohibit preterm labor
medical/surgical unit. The nurse should intervene if b. Both prohibit preterm labor and prevent
which of the following is observed? seizure
a. A nurse and client wear masks during a c. To trigger breast milk letdown
dressing change for the central catheter d. To prevent seizures
used for total parenteral nutrition
b. A nurse injects insulin through a single- 168.The nurse is caring for a client with perforated
lumen percutaneous central catheter for bowel secondary to bowel obstruction. At the time
client receiving total parenteral nutrition the diagnosis is made, which of the following should
c. A nurse applies lip balm to his/ her lips be a priority in the nursing are plan?
immediately after performing a blood draw a. Maintain the client in a supine position
to obtain specimen b. Notify the client’s next of kin
d. A nurse wears a disposable particulate c. Prepare the client for emergency surgery
respirator when administering rifampicin to d. Remove the nasogastric tube
a client withtuberculosis.
169.Which action would be the first priority when caring
161.A patient is in 8 cm dilated, 90% effaced and -3 for a client in anaphylaxis?
station when her water breaks. Immediately a. Administer an antibiotic
thereafter, the fetal heart rate decelerates in the b. Administering oxygen via face mask
60s. the nurse knows: c. Obtaining vascular access
a. This could be a sign of uterine rupture d. Preventing future antigen exposure
b. This could be a sign of cord prolapse
c. This is a normal fetal heart pattern 170.The home health nurse is performing a follow-up
d. This is a normal fetal heart pattern visit for a 76-year-old man receiving isoniazid (INH)
200 mg every day for 6 months. The nurse would be
162.Which statement by a patient with a history of major most concerned if the client made which of the
depression indicates that he is not maintaining good following statements?
health in his current environment? a. “I have blurred vision at times.”
a. “Going back to work, well, it’s not bad; it’s b. “My legs and knees hurt.”
okay.” c. “My hands and feet tingle.”
b. “I just don’t like going to the movies like I d. “I think I had a migraine yesterday.”
did before.”
c. “I can’t wait to go to my son’s wedding next 171.The nurse is obtaining a history on a client just
weekend. It will be nice to have the whole admitted to the unit. The client informs the nurse
family together.” that any information shared with the nurse during
d. “I had a great trip to the Smokey Mountains. the interview is to remain confidential. Which of the
It was fun.” following responses by the nurse is best?
a. “I’ll share any information you give me with
163.The nurse knows that which of the following mood staff members only.”
altering drugs is most often associated with an b. “If the information you share is important
increased risk for HIV infection related to to your care, I’ll need to share it with the
intravenous drug use? staff.”
a. Benzodiazepines c. “We can keep the information just between
b. Marijuana the two of us.”
c. Barbiturates d. “I have an obligation to maintain
d. Narcotics nurse/patient confidentiality about
anything you tell me.”
164.A 7-year-old child is diagnosed with insulin-
dependent diabetes mellitus. The child and parents 172.After abdominal surgery, a client is admitted from
are being taught what should occur if the child the recovery room with intravenous fluid infusing at
presents with signs and symptoms of hypoglycaemia. 100cc/hr. One hour later, the nurse finds the clamp
Which statement if made by the parents would wide open and notes that the client has received
indicate an understanding of the teaching? 850cc. The nurse would be most concerned by which
a. “It is important to decrease the amount of of the following?
long-acting insulin.” a. A CVP reading of 12 and bradycardia
b. “It is important for the child to rest in bed b. Tachycardia and hypotension
until the symptoms subside.” c. Dyspnea and oliguria
c. “It is important to increase activity prior to d. Rales and tachycardia
insulin administration.”
d. “It is important for the child to eat 4-6 173.What is the cause of blindness due to diabetic
lifesavers candies or drink orange juice.” retinopathy?
a. Haemorrhage
165.A urinalysis has been obtained on a client who has b. Tiny lesions in the tear ducts
been complaining of dysuria, urinary frequency, and c. Acidosis
discomfort in the suprapubic area. After evaluating d. Scar tissue
the results, the nurse should order a repeat
urinalysis based on which of the following findings? 174.A staff member informs the nurse that his six-year-
a. Negative glucose old child has head lice. It is most important for the
b. RBC’spresent nurse to take which of the following actions?
c. No WBCs or RBCs reported a. Inspects the staff member’s head for louse
d. Specific gravity 1.018 and nits
b. Inform the staff member that he cannot
166.The client is to have EMG. Which of the following is care for clients until further notice
the correct instruction? c. Request that the staff member contact the
a. Ask the client for allergies to seafoods physician
d. Instruct the staff member about how to use c. Teach the client that oil holds germs and
Kwell makes infection more likely
d. Wash the burn with soap and water to
175.To help prevent polypharmacy interactions in a remove the oil
client who is taking multiple prescriptions, what
instruction would the nurse give to the client? 183.The nurse is caring for a client postoperatively
a. Use a dispensing system as a reminder to following removal of a pituitary tumor. Which
take medications on a schedule observation would alert the nurse to the possible
b. Inform a family member of the names and development of diabetes insipidus (DI)?
uses of all medications a. Weight gain
c. Bring all medications, including b. Peripheral edema
supplements and herbal remedies to the c. Urinary output of greater than 200mL/hr
doctor’s appointment d. Serum sodium of 150 mEq/L
d. Abstain from taking any over-the-counter
medications in addition to the medication 184.The nurse is making a home visit for a client with an
you are already taking abdominal wound. When irrigating the draining
wound with a sterile saline solution, which of the
176.A 7-year-old client is scheduled for a cardiac following sequences would be most appropriate for
catheterization. Which priority nursing assessment the nurse to follow?
finding to report to the physician? a. Pour the solution, wash hands and remove
a. The child has an allergic reaction of hives to the soiled dressing
shellfish b. Wash hands, prepare the sterile field and
b. The child insists on taking a stuffed teddy remove soiled dressing
bear to the procedure c. Prepare the sterile field, put on sterile
c. The child has cool lower extremities with gloves, and remove the soiled dressing
brisk capillary refill bilaterally d. Remove the soiled dressing, flush the wound
d. The child has diminished palpable pedal and wash hands
pulses bilaterally

177.A 6-month-old infant has returned to the unit from 185.A client underwent a cerebral angiogram through the
surgery. Which assessment finding would indicate right femoral site. Which post-procedural nursing
that the infant was experiencing pain? assessments would justify calling the physician?
a. The child cries steadily and kicks (SATA)
b. The child points to the area producing the a. Equal, bilateral radial pulse
pain b. Bilateral pink, warm toes
c. The child has a rating of 6 on the Faces Pain c. Blood pressure of 88/52 mmHg
Rating Scale d. Pulse 122
d. The child sleeps soundly, with an increased e. Right pedal pulse weaker than left pedal
pulse rate and decreased blood pressure pulse
f. Respiration 22
178.Which of the following would the nurse see first? g. Intact dressing that needs reinforcement
a. Psychotic patient with delusion due to bloody drainage
b. Severe depression with suicide ideation
c. Patient with anxiety who is agitated 186.Which factor may contribute to the development of
d. Patient with bipolar manic phase osteoarthritis? SATA
a. Excessive use of alcohol
179.A client presents with hypoparathyroidism. Which b. 20 to 20 years of age
assessments will the nurse make with this client? c. Obesity
a. Nephrolithiasis d. Caucasian or Asian ethnicity
b. Serum calcium level of 6.8 mg/dL e. Regular strenuous exercise
c. Positive Chvostek’s sign f. Family history of osteoarthritis
d. Serum phosphorus level of 5.2mg/dL
187.An adult client is brought into the Emergency
180.In which situation is the patient most likely to Deparment in cardiac arrest. Cardiopulmonary
experience anticipatory grieving? resuscitation (CPR) is being performed. Name the
a. The patient experiences traumatic area where the pulse should be checked.
amputation of an extremity in an industrial a. Ulnar or radial pulse
accident b. Dorsalispedis pulse
b. A patient is brought into the Emergency c. Brachial pulse
Room and declared brain dead d. Carotid pulse
c. After diagnostic testing, a patient is
diagnosed with metastatic liver cancer 188.A client has a nasogastric tube in place after
d. A patient finds out that her symptoms were extensive abdominal surgery. The client complains of
from an ectopic pregnancy nausea. His abdomen is distended and there are no
bowel sounds. The first nursing action should be to:
181.A nurse notices ventricular tachycardia on the a. Administer the PRN pain medication and an
cardiac monitor at the nurse’s station and goes to antiemetic
the client’s room. What is the first action the nurse b. Irrigate the nasogastric tube with normal
should take in assisting this client? saline
a. Start cardiopulmonary resuscitation c. Determine if the nasogastric tube is patent
b. Check the patient’s airway and draining
c. Establish unresponsiveness d. Check the placement of the nasogastric
d. Alert the physician on call tube by auscultation

182.The clinic nurse observes that a ten-year-old child 189.Which emergency medication should the nurse
with leukemia has a large bum on her arm and the initially administer to a client in pulseless electrical
bum appears to be oily. The client states that she activity?
touched a hot pan and her mother put cooking fat on a. Lidocaine 4mg/min IV infusion
it so it would not blister. The nurse should: b. Atropine 1.0mg IV push
a. Document the findings in the chart c. Epinephrine 1.0mg IV push
b. Call the physician immediately to report the d. Amiodarone 400 mg IV push
injury
190.The nurse knows which of the following would have d. “You’ve been upset about your blood
the greatest impact on an elderly client’s ability to pressure.”
complete activities of daily living (ADLs)?
a. Perseveration 199.A patient undergoing hip replacement surgery who is
b. Aphasia at risk for the development of deep vein thrombosis
c. Mnemonic disturbance is receiving dalteparin (Fragmin). Which statement
d. Apraxia correctly describes the administration technique for
the medication?
191.The nurse is administering furosemide (Lasix) to a a. Use an 18-guage, 1 ½ inch needle to
patient who has edema associated with congestive administer the drug
heart failure. What is the most appropriate b. Inject the medication into the muscle
parameter for the nurse to monitor regarding within 2 inches of the umbilicus
effectiveness of this drug? c. Aspirate prior to administering the
a. Urine specific gravity medication
b. Serum potassium level d. Administer the medication by subcutaneous
c. Daily weight route
d. Abdominal girth measurement
200.The nurse is caring for patients in an acute renal
192.A 30-year-old woman is admitted to the hospital with care facility. The nurse would identify which of the
dry mucous membranes and decreased skin turgor, following patients as a likely candidate for
the woman’s vital signs are BP 120/70, temperature developing acute renal failure?
101F (38.3C), pulse 88, respirations 14. Laboratory a. A young female with recent ileostomy due
tests indicate the serum sodium is 150 mEq/L and to ulcerative colitis
Hct is 48%. The nurse expect the physician to order b. A middle-aged male with elevated
which of the following IV fluids? temperature and chronic pancreatitis
a. D5NSS c. A teenager in hypovolemic shock following a
b. 0.45% NaCl crushing injury to the chest
c. 0.9% NaCl d. Child with compound fracture on the right
d. Lactated Ringer’s femur and massive laceration of the left
arm
193.Which of the following condition are associated with
impaired glucose tolerance (IGT)?
a. Hypoglycaemia and prostatitis
b. Obesity and hypotension
c. Obesity and syndrome X
d. Hypotension and hyperlipidemia

194.You are teaching a patient who is newly diagnosed


with diabetes, how to choose healthy snacks. Of the
following foods, which is the best choice for your
patient?
a. Chocolate chip cookies with nuts
b. Ice cream
c. Buttered popcorn
d. Baked chips and salsa

195.The clinic nurse is obtaining a throat culture from a


client with pharyngitis. It is most important for the
nurse to do which of the following?
a. Quickly rub a cotton swab over both
tonsillar areas and the posterior pharynx
b. Obtain a sputum container for the client to
use
c. Irrigate with warm saline and then swab the
pharynx
d. Hyperextend the client’s head and neck for
the procedure

196.Which measures should the nursing care of a client


with hypothyroidism include?
a. Planning frequent rest periods
b. Providing cool environment
c. Encouraging the use of heating pad
d. Provide a low-calorie, high-protein diet

197.The nurse is drawing up a vitamin K injection for a


newborn. What should the dose be?
a. 1-2 mg
b. 0.5-1 mg
c. 10-50 mg
d. 12 mL

198.Which statement by a nurse in response to a patient


would be an example of a reflective question or
comment?
a. “How do you feel when you take the
medication?”
b. “Tell me what occurred first- did your
symptoms occur before or after you took
the medication?”
c. “What time do you take your medication?”
pain management using analgesics; surgery; and non-
ANSWER pharmacological therapy (focusing mainly on physical
therapy as a means of improving muscle strength to
1. C. Compulsive behavior is an unconscious attempt help control some types of pain).
to control and/or relieve the tension and anxiety the
client is experiencing It is not a manipulation on the 17. C. Fine macular rash during ciprofloxacin
client’s part. Client is not subject to depression but administration indicates hypersensitivity reaction,
to high levels of anxiety. should stop medication and notify the physician.
Option A does not warrants an immediate concern.
2. D. Infant suckling cause the posterior pituitary to Option B, PTT is within normal limits, should give
release oxytocin, which is a hormone that contracts medication. Option D, should decrease rate to
the uterus. prevent irritation of the vein.

3. A. Options B&D are symptoms of hyperthyroidism. 18. A.


Option C are symptoms of hypothyroidism
(myxedema). 19. A. Condition is often called spastic bowel disease.
Options B & C refer to inflammatory bowel disease
4. B. ECG changes can indicate potentially lethal such as ulcerative colitis or Crohn's disease. Bloody
arrhythmias such as ventricular fibrillation, which stools do not occur.
can occur in hyperkalemia.
20. D, E. Clots or fresh blood in the nose or throat,
5. A. Graves’ disease results from an increased frequent swallowing, clearing of the throat, and
production of thyroid hormone. It is state of vomiting of dark blood are indications of possible
hypermetabolism. The increased metabolic rate bleeding. Check the back of the patient's throat with
generates heat and produces tachycardia and fine a flashlight for trickling of blood. Decreased BP,
muscle tremors. Patients are encourage to have tachycardia, pallor, and restlessness are hallmark
frequent rest periods, and are advised to avoid signs of hemorrhage and should be reported to the
strenuous physical activity. Management include the surgeon immediately.
use of antithyroid drugs (propylthiouracil or
Tapazole), radioactive iodine, or surgical removal of 21. C. Carvedilol is a nonselective beta-adrenergic
a portion of the gland. antagonist that blocks the action of beta1 receptors
in the heart and the action of beta2 receptors in the
6. C. Early signs of lithium toxicity are Fine tremors, lung, smooth and skeletal muscles. Blocking the
nausea, vomiting, diarrhea. Late signs include beta1 receptor leads to deacreased heart rate,
Ataxia, confusion, seizures contractility and velocity of impulse conduction in
the atrioventricular node. Beta2 receptors blockade
7. A, D, E, F. The presence of frankly bloody emesis can result in bronchoconstriction and inhibition of
(hematemesis) suggests moderate to severe bleeding glycogenolysis. Because of this drug’s effect on the
that may be ongoing, whereas coffee-ground emesis heart, the nurse should assess the patient’s current
suggests more limited bleeding. Melena may be seen pulse and blood pressure before administering
with variable degrees of blood loss, being seen with carvedilol. The prescriber should be contacted if
as little as 50 mL of blood. Hematochezia (red or bradycardia or hypotension is identified prior to
maroon blood in the stool) is usually due to lower GI administration of the drug. Carvedilol is
bleeding. However, it can occur with massive upper administered orally.
GI bleeding, which is typically associated with
orthostatic hypotension. 22. B. First signs of acute rejection are usually a rash,
burning, and redness of the skin on the palms and
8. B. soles. This can spread over the entire body. Other
symptoms include nausea, vomiting, stomach
9. B. Consolidation will result in diminished breath cramps, diarrhea (watery and sometimes bloody),
sounds over the lobes involved. Wheezing results loss of appetite, yellowing of the skin and eyes
from constricted airways such as in asthma. (jaundice), abdominal (belly) pain, weight loss.
Bronchovesicular breath sounds are normal lung
sound. Hyperresonance results from percussing an 23. A.
excessively air-filled lung or pleural space.
24. D. Muscles are generally strongest in the morning,
10. C. Atropine has a vagolytic effect as well as blocks and activities involving muscle activity should be
muscarinic responses and has selective depression of scheduled then. There is no decrease in sensation
central nervous system. Benadryl is an H-1 receptor with MG, and muscle atrophy does not occur because
antagonist and antihistamine with anticholinergic muscles are used during part of the day.
activity and does not protect against vagal
bradycardia. Adrenalin is a catecholamine that 25. B.
constricts bronchioles and inhibits histamine release,
and Prozac is a antidepressant.
26. B. Magnesium sulfate is a central bervous system
11. D. depressant and relaxes smooth muscles. Adverse
effects include flushing, depressed respirations,
12. C. The suicide of her son puts this patient at high depressed deep tendon reflexes, hypotension,
risk of suicide. This risk is exacerbated by the extreme muscle weakness, decreased urinary
betrayal of her husband and best friend. output, pulmonary edema and elevated serum
magnesium levels.
13. C. *
27. B. Stress ulcers or Curling's ulcers are acute
ulcerations of the stomach or duodenum that form
14. C. Notify the if there is arm or hand numbness, following the burn injury.
coldness, tingling, swelling, or pain
28. C. Olanzapine (Zyprexa) is an atypical antipsychotic
15. C. drug given to patients with schizophrenia. Key
treatments for obsessive-compulsive are
16. A. Four main methods of treatment exist for a benzodiazepines and SSRIs. Fluoxetine (Prozac) and
patient with Paget’s disease, pharmacological other SSRIs are given to patients with bulimia
therapy using either bisphosphonates or calcitonin;
nervosa. Seroquel is an atypical antipsychotic. CSF lost during test and to flush out the dye. Option
Buspar is an anxiolytic drug. C is appropriate for MRI.

29. B. Raynaud’s disease is characterized by attacks of 44. D. Client is under constant observation; must not be
vasospasms in the small arteries and arterioles of the left alone for any reason
fingers and sometimes the toes. The disease
45. C.
primarily affects young women and can be triggered
by exposure to cold. 46. C. Stridor on exhalation is a hallmark of respiratory
distress, usually caused by obstruction resulting from
30. D. Utilization of an MDI requires coordination edema. One emergency measure is to remove the
between activation and inspiration; deep breaths to surgical clips to relieve the pressure. In some
ensure that medication is distributed into the lungs, settings, this may be a nursing action; in other
holding the breath for 10 seconds or as long as settings, this is a physician function. Emergency
possible to disperse the medication into the lungs, intubation also may be necessary.
shaking up the medication in the MDI before use, and
a sufficient amount of time between puffs to provide 47. A.
an adequate amount of inhalation medication.
48. B. Option A & D, reveals normal values: BUN 7–18
mg/dL; creatine 0.6–1.2 mg/dL; AST (formerly
31. C.
SGOT) 8–20 U/L, ALT (formerly SGPT) 8–20 U/L,
bilirubin 0.1–1.0 mg/dL . Amphotericin B causes
32. B. renal and liver toxicity. Check liver and renal
function studies weekly, notify the physician if
33. C. elevated. Option C are not side effects of the
medication.
34. B. The pathology behind PIH is a fluid shift that
occurs from the vasculature to the tissues, which 49. B.
causes edema and leads to an increase in
hematocrit. 50. D.

51. A, C, F.
35. A.
52. A. Robust means strong and healthy. Pulse pressure
36. C. of 40 is normal. All other options reveal abnormal
assessment.
37. A. SIADH is a condition in which the client has
excessive levels of antidiuretic hormone (ADH) and 53. C, E, F.
can’t excrete the diluted urine. Therefore, the
client retains fluids. This disorder causes a dilutional 54. C. This patient has experienced a loss (job) that is
hyponatremia. contributing to his feelings of uselessness to his
family. The diagnosis of situational low self-esteem
is the most appropriate diagnosis for this patient.
38. A.
The North American Nursing Diagnosis Association
(NANDA) definition for the nursing diagnosis is the
39. A, C, E. An endoscopic retrograde development of a negative perception of self-worth
cholangiopancreatography is anendoscopic test that in response to a current situation.
provides radiographic visualization of the bile and
pancreatic ducts. Postprocedural care after the ERCP 55. B. Antipyretics relieve the combination of side
include monitoring the vital signs and maintaining an effects. Ice bag is dangerous to both skin integrity
NPO status until the gag reflex returns. The client and overall temperature control. Option D is
probably received sedating medication before the unnecessary unless indicated for another reason.
procedure; consequently, lethargy is expected. A
local anesthetic is sprayed into the client’s throat, 56. D. Cushing's triad is systolic hypertension with a
so it is possible that the gag and cough reflexes will widening pulse pressure, bradycardia with a full and
not be present. The client should be monitored for bounding pulse, and slowed respirations. The rise in
signs of cholangitis and perforation, which include blood pressure is an attempt to maintain cerebral
perfusion, and it is a neurologic emergency because
fever, abdominal pain (especially in the RUQ),
decompensation is imminent.
hypotension and tachycardia.
57. C. Several strains of the human papillomavirus (HPV)
40. D. Ibandronate (Boniva) is a bisphosphonate drug. In are associated with cervical cancer. Antibiotics are
giving such drugs the nurse should instruct the client not used for viral infections. Douches will not
to swallow the whole tablet, it should not be prevent recurrence of the disease. Tampons would
chewed. It should be taken in the morning on an not be a problem as in toxic shock syndrome.
empty stomach with large glass of water (6-8oz) and
wait at last 30 minutes before eating or lying down. 58. B.
Make sure the client has adequate intake of Vitamin
D. Instruct the client to report any signs and 59. B.
symptoms of gastric reflex or pain.
60. B. Environments with increased numbers of children
(day care) more likely to promote infections due to
41. B.
close living conditions and increased likelihood of
42. C. The airway is compromised by the bleeding in the disease transmission. Option A & D do not pose a
esophagus and aspiration easily occurs. problem or solution regarding gastroenteritis.
Neighbourhood is a possible source of infection, but
43. A. Dye is injected into subarachnoid space before an not as likely as a day care center.
x-ray of spinal cord and vertebral column to assist in
identifying spinal lesions; if client is allergic to dye, 61. A.
there is a major safety issue. It is important that
client drink extra fluids AFTER the test to replace 62. B. Position of the trachea should be evaluated; with
a tracheal shift, an increase in pressure could occur
on the operative side and could cause pressure on the right arm. The patient is taught to use the
against the mediastinal area. On the surgical side, PCA as needed for pain control rather than at a set
breath sounds will be absent. Sputum is important to time. The nurse allows the patient to examine the
observe but not as high a priority. Pulse pressure incision and participate in care when the patient
does not relate to the situation feels ready. Permanent breast prostheses are usually
obtained about 6 weeks after surgery.
63. A.
88. C.
64. D.
89. A. After a total hip replacement, it is important to
65. C. maintain the hip in a state of abduction to prevent
dislocation of the prosthesis. Use of an abduction
66. D. pillow or splint will not prevent the formation of
sacral pressure ulcers. An abduction pillow may also
67. A. Potassium chloride must be diluted and be used to keep the legs shoulder width apart and to
administered at a rate no faster than 20mEq/hr prevent rotation of the hips, and avoid crossing the
leg beyond the midline of the body (e.g. not crossing
68. C. Nitroglycerin patch should be removed before the the leg over the other leg).
test. Dentures are removed removed before the test.
Option B, results are within normal limits. It is
unnecessary check capillary refill. 90. A. Among the 4 women, the least stable patient is
the patient experiencing unilateral dull abdominal
69. A. Delusion of perseceution is a strongly held belief pain. The pregnant woman needs to be evaluated
that is not validated bu reality, for example, the immediately for ectopic pregnancy. Option B is
idea that his brother is trying to steal his property is expected during first trimester of pregnancy. Option
not validated by reality. Delusion of reference is a C is symptomatic of threatened abortion, should be
false belief that public events or people are directly instructed to decrease activity. Option D symptoms
related to the individual. suggest of spontaneous abortion and should be
instruct client to pads. Ectopic pregnancy needs
70. C. more emergent intervention as compared to
abortion.
71. A.
91. C. Instillation of erythromycin into the neonate’s
eyes provides prophylaxis for opyhalmia neonatorum
72. C. During a sickle cell crisis, increasing the transport or neonatal blindness caused by gonorrhoea in the
and availability of oxygen to the body's tissues is mother. Erythromycin is also eefctive in the
paramount. Administering a high volume of prevention of infection and conjunctivitis from
intravenous fluid and electrolytes to help Chlamydia trachomatis. The medication may result
compensate for the acidosis resulting from in redness of the neonate’s eyes, but this redness
hypoxemia associated with sickle cell crisis is one will eventually disappear. Erythromycin ointment is
way to accomplish this. Fluid administration also not effective in treating neonatal chorioretinitis
helps overcome dehydration, a possible predisposing from cytomegalovirus. No effective treatment is
factor common in clients with sickle cell crisis. available for a mother with cytomegalovirus.

73. D. 92. B.

74. C. 93. D. EKG changes in hyperkalemia: Peaked T waves;


Widened QRS complexes; prolonged PR intervals;
75. B. Extrapyramidal reactions include movement Flat P waves
disorders such as dystonia, tardive dyskinesia, and
pseudoparkinsonism. 94. B.

76. A. 95. B.

77. A. 96. C.

78. D. 97. D. The temperature range during which the basal


metabolic rate of the baby is at a minimum, oxygen
79. D. utilization is least and baby thrives well is known as
'Thermo- neutral range of temperature' or 'Neutral
80. B. Thermal Environment'. For each baby, this range of
temperature varies depending on gestational age.

81. A. ECG monitoring of acute coronary syndrome:


features that increase the likelihood of infarction 98. A. first dose of the DPT may be given at two months,
are: new ST-segment elevation; new Q waves; any the second is given around four months. MMR is given
ST-segment elevation; new conduction defect. Other at 15 months. Polio is given at two and four months
features of ischaemia are ST-segment depression and and again at 12 to 18 months. Smallpox vaccine is no
T-wave inversion. longer recommended.

82. A.

83. A.

84. B.

85. B.

86. D.

87. D. The patient is at risk for lymphedema and


infection if blood pressures or venipuncture are done
117.D. A goal when caring for patients with spinal cord
99. D. Diagnosis of rheumatic fever is based on the Jones injuries is to promote their adjustment to the injury
criteria and positive laboratoty tests for: Increased and their independence. A patient who is using
erythrocyte sedimentation rate; positive C-reactive modified feeding utensils at every meal is
protein; positive antistreptolysin-O titer; positive demonstrating an attempt at independence for the
throat culture for group A beta haemolytic functional activity of eating. The patient's statement
streptococci; prolonged PR and QT intervals, recognizes that the activity is one that requires
revealed by ECG. continued work, but progress is being made toward
the goal of developing as much independence as
100. B. The absence of spontaneous cough and presence possible with eating.
of drooling and agitation are cardinal signs
distinctive of epiglottitis. 118.C. Other options suggest exacerbation of the
disease.
101.B. Hallmark of aseptic technique is handwashing.
Client should use only the prescribed medications on 119.A.
the wound.
120.D. Water should not be allowed to stand in
102.D. containers, such as respiratory or suction equipment
because this could act as a culture medium.
103.B. Take the drug with food or after meals if GI upset
occurs. Do not cut, crush, or chew sustained-release 121.A. Neurogenic shock is characterized by areflexia. In
products. Report ringing in the ears; dizziness, neurogenic shock, vasodilation occurs as a result of a
confusion; abdominal pain; rapid or difficult loss of balance between parasympathetic and
breathing; nausea, vomiting, bloody stools, easy sympathetic stimualtaion. The patient experiences a
bruising, gum bleeding (related to aspirin’s effects predominant parasympathetic stimulationthat causes
on blood clotting). vasodilation lasting for an extended period. It is
manifested by hypotension, due to a reduction in
104.B. Fontanelle should feel soft and flat; fullness or systemic vascular resistance and venous return;
bulging indicates increased intracranial pressure. warm, dry skin; bradycardia, due to dominance of
Umbilical cord falls off within 1 to 2 weeks; no tub the parasympathetic system (vagus nerve); flaccid
baths until the cord falls off. Normal healing of paralysis, including bowel and bladder; hypothermia,
cirucmcision, don't remove exudate; clean with due to vasodilation.
warm water. motor reflex is normal; disappears
after 3 to 4 months. 122.C.

105.D. 123.B. By administering Librium, you will prevent


delirium tremens that can possibly harm during the
106.C. Principles for radiation therapy are time, process.
distance, shielding; nurse should decrease the time
spent in close proximity to the client. All visitors are 124.A, D, E, F.
restricted with regard to the distance they should be
from the client. Appropriate shielding (lead aprons) 125.A. Open Kaposi’s sarcoma lesions should be cleaned
is to be used when the nurse has to spend any length and dressed daily to prevent secondary infection.
of time at a close distance, not just for routine care. Warm tub bath is not done because of risk of
secondary skin infection. It is important to keep the
107.C. skin clean to prevent secondary skin infection but
should be covered due to open areas. Diluted
108.C. Nystatin is given for candida infections such as povidone-iodine is the treatment for herpes simplex
oral thrush. A common side effect is diarrhea. virus abscess, not Kaposi’s sarcoma.

109.B. 126.C. Assessment of the dressing should be checked on


admission to the room and frequently for the next
several hours. Option A is an Implementation,
110.A. Signs and symptoms of abdominal aortic aneurysm complete assessment first. Option B is an assessment
include: diminished femoral pulses, lower back and but determine what is happening to the patient
abdominal pain, pulsatile abdominal mass, bruit over now. Option D is an inappropriate assessment, it is
site, BP difference between extremities, peripheral too soon for infection to occur secondary to surgery.
ischemia.
127.C. Polio vaccine contains live virus and should not be
111.D. Patient is at risk for peritonitis, and should be given to children who are immunocompromised.
assessed for further symptoms of infection. Option A
reveals small amount of bleeding and does not 128.A. Unsteady gait, slurred speech, nausea, vomiting,
indicate acute bleeding. Option B suggests expected diarrhea, thirst, and coarsening of hand tremors
outcome. Option C indicates resolution. indicate lithium toxicity.

112.A. Ostomy bags should be changed at least once a 129.D. Hypertensive crisis, an adverse effect of this
week or when seal arpunf stoma is loose or leaking; medication, is characterized by hypertension,
during change of bag isa good time for stoma and frontally radiating occipital headache, neck stiffness
skin to be closely inspected. Client should be and soreness, nausea, vomiting, sweating, fever and
encouraged to participate and should foster chills, clammy skin, dilated pupils, and palpitations.
independence. Tachycardia, bradycardia, and constricting chest
pain may also be present. The client is taught to be
113.A. alert to any occipital headache radiating frontally
and neck stiffness or soreness, which could be the
114.A. first signs of a hypertensive crisis. Hypertensive crisis
is treated with (Nifedipine) Procardia.
115.A. Patient has not lost civil right to use phone.
Patient is able to use phone unless otherwise 130.D.
indicated by court order or physician’s order.
116.D. Garlic, a lipid lowering-agent, increase bleeding 131.A. Tracheostomy set is the most important for the
potential with aspirin, NSAIDs and warfarin. client's safety due to risk for laryngospasm.
supportive to teach clients how to deal with the
132.B. traumatizing situation and the emotional aftermath.
Other options are reasonable recommendations to
133.D. begin using in a systematic desensitization program
after the crisis is alleviated.
134.C. Extrapyramidal side effects resulting from this
medication include akathisia (motor restlessness), 152.C. Vitamin K food sources are green leafy
dystonias (protrusion of tongue, abnormal vegetables, cauliflower and cabbage.
posturing), pseudoparkinsonism (tremors, rigidity),
and dyskinesia (stiff neck, difficulty swallowing). The 153. C. The nurse must immediately call and
dosage may need to be decreased because of side report the suspected abuse. Failure to report abuse
effect of medication; antiparkinsonian drug such as is a misdemeanor.
Cogentin may be ordered. Option B is not accurat,
Thorazine is an antipsychotic medication. 154.C.

135.D. 155.D. Milking forces interstitial fluid to mix with


capillary blood and dilutes the blood. Dangling helps
136.B. You should not cut extended-release tablets in facilitates venous congestion. Sticking on the side is
half. less painful that the center of the fingertip. Blood
should sit on the strip like a raindrop, smearing
137. A. The patient may be experiencing hypocalcemia. alters the reading.

138.A, C, D, E. 156.C. Use the least restrictive interventions in


ascending order.
139.C.
157.D. Acute phase of burn injury occurs from beginning
140.B. Postural hypotension can be a result of dizziness of diuresis to the near completion of wound closure
owing to the use of low-potency antipsychotics such Characterized by fluid shift from interstitial to
as chlorpromazine or thioridazine. intravascular. Urinary output of less than 30ml/hr
should reported to the physician.
141.A,B,C,E. Hemolytic Reaction is a type of
complication of blood transfusion is caused by 158.C. 1 milligram = 1000 mcg
infusion of incompatible blood products. Assessment
include Low back pain (first sign). This is due to 159.B. In HHNS, CNS depression, disorientation or mental
inflammatory response of the kidneys to confusion, seizures, and coma are caused by
incompatible blood; Chills; Feeling of fullness; intracellular dehydration and hyperosmolarity. CNS
Tachycardia; Flushing; Tachypnea; Hypotension; dysfunction worsens as serum osmolarity rises.
Bleeding; Vascular collapse; Acute renal failure.
160.C. Applying lip balm or handling contact lenses is
142.D. Droplet precautions used for organisms that can prohibited in work areas where exposure to
be transmitted by face-to-face contact, door may bloodborne pathogens may occur. Option A
remain open. Option A&B require contact demonstrates appropriate procedure, prevents
precautions with no mask. Option C requires airborne contamination. Insulin is the only
standard precautions. medication that can be given, compatible with TPN.
Use airborne precautions for TB, private room with
143.C. Naloxone is a narcotic antagonist that can reverse negative air pressure, minimum of six exchanges per
the effects, both adverse and therapeutic, of opioid hour.
narcotic analgesics. Morphine is an opioid narcotic
analgesic that can depress respiration. 161.B.

144.B. 162.B. Anhedonia—the loss of interest and pleasure in


activities—is a sign of depression.
145.A. To implement a behavior modification plan
successfully, all staff members need to be included 163.D. Narcotics are most often used intravenously
in program development, and time must be allowed
for discussion of concerns from each nursing staff
member; consistency and follow-through is 164.D. Candy or another simple sugar is carried and used
important to prevent or diminish the level of to treat mild hypoglycemia episodes.
manipulation by the staff or client during
implementation of this program. 165.C. With the client’s complaints, WBCs and RBCs
should be present; WBCs are a response to the
146.A. inflammation process and irritation of the urethra;
RBCs are increased when bladder mucosa is irritated
and bleeding. Glucose increases during the
147.B inflammation process; it is not a primary component
in determining urinary tract infections.
148.C.
166.A.
149.C. Alteplase is a tissue plasminogen activator which (B.) Electromyography (EMG) assesses electrical
induce fibrinolysis that causes bleeding. activity associated with nerves and skeletal muscles.
Needle electrodes are inserted to detect muscle and
150.A. Lamotrigine, marketed in the US and most of peripheral nerve disease. You should inform the
Europe as Lamictal by GlaxoSmithKline, is an patient that pain and discomfort are associated with
anticonvulsant drug used in the treatment of insertion of needles. There is no risk of electric
epilepsy and bipolar disorder. It is also used off-label shock with this procedure.
as an adjunct in treating depression. For epilepsy, it
is used to treat focal seizures, primary and 167.B. Magnesium Sulfate is used to reduce preterm
secondary tonic-clonic seizures, and seizures labor contractions and prevent seizures in Pre-
associated with Lennox-Gastautsyndrome. Eclampsia

151.C. Support groups of people who have suffered 168.C.


similar acts of violence can be helpful and
169.B. carried out after the patency of the tube is
determined. Patency should be checked first by
170.C. Isoniazid/INH can causes peripheral neuritis. Vit aspirating stomach contents, not by auscultation.
B6/Pyridoxine is given.
189.C. The mainstay of drug therapy for PEA is
171.B. The nurse is obligated to share client information epinephrine 1 mg every 3–5 minutes.
with personnel directly involved with the client’s
care. The nurse must never agree to keep 190.D.
information confidential without knowing the
content of the information. The nurse is not 191.C.
obligated to report information that is not relevant
to the client’s care or well-being. 192.B. Hypotonic solution, shifts fluid into intracellular
space to correct dehydration. Hypertonic solutions
172.D. Indicates cardiovascular fluid overload. are contraindicated in dehydration. Isotonic solutios
are not best with dehydration. Lactated Ringer’s is
173.A. Diabetic retinopathy leads to development of an isotonic solution used to replace electrolytes.
microaneurysms and intraretinal haemorrhage.
193.C.
174.A. Observe for movement (louse) or small whitish
oval specks that adhere to the hair shaft (nits); treat 194.D.
with gamma-benzene hexachloride (Kwell). Confirm
the presence of lice before excluding from duty; if 195.A. To obtain a throat culture specimen, the nurse
lice present, exclude from patient care until puts on clean gloves, then inserts the swab into the
appropriate treatment has been received and shown oropharynx and runs the swab along the tonsils and
to be effective. Assessment should be done first. areas on the pharynx that are reddened or contain
Apply Kwell shampoo to dry hair and work into lather exudate. The gag reflex, active in some clients, may
for 4–5 minutes be decreased by having the client sit upright if
health permits, open the mouth, extend the tongue,
175.C. Polypharmacy means that multiple medications and say "ah," and by taking the specimen quickly.
have been prescribed.
196.D.
176.A. People who have an allergy to shellfish or iodine
may experience an allergic reaction to the contrast 197.B.
dye.
198.D. The statement "You've been upset about your
177.A. blood pressure" is a reflective comment that
describes the patient's feelings. A reflective
178.B. comment repeats what a patient has said or
describes the person's feelings. It is used by the
179.C. nurse to encourage the patient to elaborate on the
topic.
180.C. Grief can be classified as acute, anticipatory, or
pathologic. Anticipatory grief is associated with the 199.D. Dalteparin is given by subcutaneous (under the
anticipation of a death or loss that has yet to take skin) injection, usually in the lower abdominal area.
place. A patient who is newly diagnosed with liver
cancer is most likely to experience anticipatory 200.C.
grieving when anticipating death.

181.C.

182.D. Because leukemic clients are immunosuppressed,


they are more susceptible to infections; cooking fat
applied to an open wound increases the possibility of
infection; burns should be rinsed immediately with
tap water to reduce the heat in the burn.
Documentation is done later, and does not address
the immediate problem of cleansing the wound.
Reporting the physician is unnecessary unless signs of
wound infection is noted.

183.C.

184.B. Handwashing should be done prior to beginning


any procedure, especially irrigating a wound. Using
sterile gloves to remove the dressing would
contaminate them.

185.D,E,G.

186.C,D,F.

187.D. During CPR, the carotid artery pulse is the most


accessible and may persist when the peripheral
pulses (radial and brachial) no longer are palpable
because of decreases in cardiac output and
peripheral perfusion.

188.C. Should first assess if the tube is open and draining


to determine if there is a problem with the
nasogastric tube; if it is patent and draining it does
not need to be irrigated. Option A & B, may be

You might also like